An Image point after a 180° rotation Is Z(3, 7). What were the coordinates of the pre-Image point?
Z-3-7)
2173)
27-3)
21-7-3)

An Image Point After A 180 Rotation Is Z(3, 7). What Were The Coordinates Of The Pre-Image Point?Z-3-7)2173)27-3)21-7-3)

Answers

Answer 1

Answer:

Does the answer help you?

An Image Point After A 180 Rotation Is Z(3, 7). What Were The Coordinates Of The Pre-Image Point?Z-3-7)2173)27-3)21-7-3)

Related Questions

Which of the following questions are equivalent to the answer below x 3/5

Answers

Answer:

[tex]x^\frac{3}{5} = (x^3 )^\frac{1}{5}[/tex]

[tex]x^\frac{3}{5} = \sqrt[5]{x^3}[/tex]

[tex]x^\frac{3}{5} = (\sqrt[5]{x})^3[/tex]

Step-by-step explanation:

Given

[tex]x^\frac{3}{5}[/tex]

Required

The equivalent expressions

We have:

[tex]x^\frac{3}{5}[/tex]

Expand the exponent

[tex]x^\frac{3}{5} = x^{ 3 * \frac{1}{5}}[/tex]

So, we have:

[tex]x^\frac{3}{5} = (x^3 )^\frac{1}{5}[/tex] ----- this is equivalent

Express 1/5 as roots (law of indices)

[tex]x^\frac{3}{5} = \sqrt[5]{x^3}[/tex] ------ this is equivalent

The above can be rewritten as:

[tex]x^\frac{3}{5} = (\sqrt[5]{x})^3[/tex] ------ this is equivalent

Which graph shows a set of ordered pairs that represent a function?

Answers

Answer:

Graph C.

*See attachment below

Step-by-step explanation:

A graph that shows a set of ordered pairs representing a function would have each x-value being plotted against only one y-value. That is, every x-value must have exactly one y-value. Every x-value must not have more than 1 y-value being plotted against it.

The graph that shows this is the graph in option as shown in the attachment below.

The College Board conducted research studies to estimate the mean SAT score in 2016 and its standard deviation. The estimated mean was 1020 points out of 1600 possible points, and the estimated standard deviation was 192 points. Assume SAT scores follow a normal distribution. Using the Empirical Rule, about 95% of the scores lie between which two values?
a. 768 to 1358
b. 636 to 1404
c. 620 to 1520
d. 828 to 1212
e. 724 to 1486

Answers

Answer:

its B. 636 to 1404

Step-by-step explanation:

Using the Empirical Rule, about 95% of the scores lie between values 636 to 1404. The correct option is c.

What is standard deviation?

The standard deviation of a set of values is a measure of its variation or dispersion. The square root of the variance, which is the average of the squared differences from the mean, is used to calculate it.

According to the Empirical Rule, approximately 68% of the data for a normal distribution fall within one standard deviation of the mean, 95% within two standard deviations, and 99.7% within three standard deviations.

In this case, the mean SAT score is 1020, with a standard deviation of 192. As a result, roughly 95% of the scores fall within two standard deviations of the mean, or between

(1020 - 2(192)) and (1020 + 2(192)).

Calculating, we get:

Lower bound: 1020 - 2(192) = 636

Upper bound: 1020 + 2(192) = 1404

Therefore, the answer is (b) 636 to 1404.

For more details regarding standard deviation, visit:

https://brainly.com/question/23907081

#SPJ2

If two angles are complementary, find the measure of each of angle.

Answers

Answer:

B: 30 and 60

Step-by-step explanation:

First, let's set up an equation. Since the two angles are complementary, we can write the equation like this:

2p + p = 90

Now, let's solve it!

2p + p = 90

Combine like terms:

3p = 90

Divide each side by 3 to isolate p:

3p/3 = 90/3

p = 30

Now that we know how many degrees one of our angles is, we can subtract that from 90 to get both of the complementary angles.

90 - 30 = 60

Therefore, the two angles that are complementary in this case are 30 and 60 degrees.

Please find the answer

Answers

9514 1404 393

Answer:

  -0.84

Step-by-step explanation:

In decimal, the expression is ...

  -1.34 +1.50 -1.00 = -0.84

__

As fractions, the expression is ...

  -67/50 +75/50 -50/50 = (-67+75-50)/50 = -42/50 = -21/25

I’m having trouble with this

Answers

Answer:

this will give you the answer: for cylinder

V = 3×2^2×7 = 84cm

this will give you the answer for cone:

V = 3× 2^2 × 6/3 = 24cm

then we just add

84 + 24 = 108cm^3

Step-by-step explanation:

hope it helps!

A survey asked 50 students if they play an instrument and if they are in band.
1.25 students play an instrument.
2. 20 students are in band.
3. 30 students are not in band.
Which table shows these data correctly entered in a two-way frequency?

Answers

C, just look at the "Total" for each single information.

the values in the inner grid combine multiple informations.

The table shows these data correctly entered in a two-way frequency is table C.

What is Two way Frequency?

Two-way frequency tables show the potential connections between two sets of categorical data visually. The table's four (or more) inside cells contain the frequency (count) data, which is displayed above and to the left of the table's designated categories.

We have been the information 25 students play an instrument 20 are in a band 30 are not in a band.

So, the two way table is:

                                     Band            Not in Band       Total

Play instrument                20                   5                     25

Do not play instrument     0                     25                 25

Total                                  20                    30                50

So, Table C is Correct.

Learn more about two-way frequency here:

https://brainly.com/question/9033726

#SPJ7

Solve for y.

Z ​= yn

Answers

Answer:

y = z /n

Step-by-step explanation:

Answer:

y=z/n

Step-by-step explanation:

To isolate the y, divide both sides by n

The height of a triangle is 2 times the base. The area is 4 square inches. Find the base.
The base of the triangle is
inches.

Answers

Answer:

2 Inches

Step-by-step explanation:

Area of a triangle = (1/2)* Base * Hight

lets consider the base of the triangle is X inches,

then, Hight of the triangle is 2X

Then the Area of the Angle is = (1/2)*X*2X

                                               4 = x^{2}

                                               X = 2

find the value of the trigonometric ratio​

Answers

Answer:

ur box cannot be opend  repain the window

Step-by-step explanation:

sin A=opposite / hypotenusesin A=32/40 orsin A=32/40 (divide a number by 2)sin A=9/5

please mark this answer as brainlist

tiếp tuyến của đồ thị hàm số x[tex]x^{3} -3x^{2} +2[/tex] tại điểm M(2,-2)

Answers

Answer:

what are you telling Id understand

Step-by-step explanation:

became

square root 12 is ___ greater than square root 7

Answers

It’s gotta be 1.whatever

Answer:0.8

Step-by-step explanation:

Please help
A. SAS
B. AAS
C. HA
D. LL
E. ASA
F. HL

Answers

Answer:

Option B, AAS

Option C, HA

Option E, ASA

these three options applies

9.2% written as a decimal is

Answers

the answer will be 0.092 as a decimal

Find the lengths of the other two sides of the isosceles right triangle

Answers

Answer:

[tex]x=5[/tex]

[tex]h=\sqrt{(5)^{2}+x^{2} } =\sqrt{(5)^{2}+(5)^{2} }[/tex]

[tex]h=\sqrt{25+25} =\sqrt{50}[/tex]

[tex]h=5\sqrt{2}[/tex]

OAmalOHopeO

the length of a building is 720 in. Use dimensional analysis to convert this quantity to yards.

Answers

Answer:

20 yards

Step-by-step explanation:

First, we have to know how to get from inches to yards. 12 inches = 1 foot, and 3 feet = 1 yard.

Next, we have

[tex]\frac{720 in}{1}[/tex]

as a given. We want to multiply this by our conversions to get the end amount in yards.

We can first multiply our expression by 12 inches = 1 foot. To do this, we can set one number as the divisor and one as the dividend, e.g. 12 inches /  1 foot. However, to figure out which number should be the numerator/denominator, we first must examine our original expression. We want to multiply it by a value that cancels something out in conversion. To do this, we can have the same unit of something in the numerator in one and the denominator in the other in terms of what we're multiplying. Here, we can see that [tex]\frac{720 in}{1}[/tex] has inches on the top, so we want inches on the bottom in 12 inches = 1 foot. Therefore, we have

[tex]\frac{720 in}{1} * \frac{1 foot}{12 inches}[/tex]

= [tex]\frac{720feet}{12}[/tex]

= 60 feet

= [tex]\frac{60 feet }{1}[/tex]

We have feet as the numerator, so we want feet in the denominator in what we're multiplying this by. In 3 feet = 1 yard, we can multiply this to get

[tex]\frac{60 feet }{1} * \frac{1 yard}{3 feet} \\= \frac{60 yards}{3}[/tex]

= 20 yards

Express the area of the entire rectangle.
Your answer should be a polynomial in standard form.

please answer quick i need to go to my friends to get my joy con fixed

Answers

The area is just the base times the height. In this case, the base is (x+4) and the height is (x+6), and then you just distribute to get x^2 +4x+6x+24 which is x^2+10x+24.

20. simplify each of the following: see the above picture
and get 40 points

Answers

Answer:

[tex]i)14 + 4 \sqrt{6} [/tex]

[tex]ii) \sqrt{10} + 28[/tex]

[tex]iii) 243[/tex]

Step-by-step explanation:

[tex]i)(2 \sqrt{3} + \sqrt{2} {)}^{2} [/tex]

➡️ [tex]12 + 4 \sqrt{6} + 2[/tex]

➡️ [tex]14 + 4 \sqrt{6} [/tex] ✅

●●●●●●●●●●●●●●●●●●●●●●●●●●●●●●

[tex]ii)(3 \sqrt{5} - \sqrt{2} ) \times ( \sqrt{2} + 2 \sqrt{5} )[/tex]

➡️ [tex]3 \sqrt{10} + 30 - 2 - 2 \sqrt{10} [/tex]

➡️ [tex] \sqrt{10} + 28[/tex] ✅

●●●●●●●●●●●●●●●●●●●●●●●●●●●●●●

[tex]iii)3 \sqrt{81} \times 3 \sqrt{9} [/tex]

➡️ [tex]3 \times 9 \times 3 \times 3[/tex]

➡️ [tex]243[/tex] ✅

please solve asap thanks ​

Answers

Answer:

6x+36=180

6x=144

x=24

Step-by-step explanation:

this is the correct answer

X=24 That is the correct answer

11. A surveyor at point S discovers that the angle between peaks A and B is 3 times as large as the angle
between peaks B and C. The surveyor knows that ZASC is a right angle. Find mzASs and m2BSC.

Answers

The measures of the angles between the peaks are;

m∠BSC = 22.5°

m∠ASB = 67.5°

The reason for arriving at the above angles is as follows:

The known values are;

The location of the surveyor = Point S

The angle between peaks A and B = m∠ASB = 3 times as large as the angle between peaks B and C = 3 × m∠BSC

The measure of angle m∠ASC = A right angle = 90°

Required:

To find m∠ASB and m∠BSC

From the given diagram, we have;

m∠ASC = 90°

m∠ASC = m∠ASB + m∠BSC (angle addition postulate)

m∠ASB = 3 × m∠BSC

∴ m∠ASC = 3 × m∠BSC + m∠BSC = 4 × m∠BSC

m∠ASC = 4 × m∠BSC = 90°

m∠BSC = 90°/4 = 22.5°

m∠BSC = 22.5°

m∠ASB = 3 × m∠BSC

∴ m∠ASB = 3 × 22.5° = 67.5°

m∠ASB = 67.5°

Learn more about angle addition postulate here:

https://brainly.com/question/4208193

square root of v-5=6
[tex] \sqrt{x - 5 = 6}[/tex]

Answers

Answer:

I think you mean this :

[tex] \sqrt{x - 5} = 6 \\ = > {( \sqrt{x - 5} })^{2} = {6}^{2} \\ = > x - 5 = 36 \\ = > x = 36 + 5 \\ = > x = 41[/tex]

Or,

Square root of x-5=6 is :

[tex] \sqrt{x - 5} \:=\:\sqrt{6} [/tex]

What is the slope of the line that passes through the points listed in the table?
x | y
4 | 7
5 | 3

A. 4
B. 3
C. -3
D. -4

Answers

Answer:

D. -4

Step-by-step explanation:

the slope formula is

m=(y2-y1)/(x2-x1)

(x2,y2) = (4,7)

(x1, y1) = (5,3)

So (7-3)/(4-5) = 4/-1 = -4

Answer:

-4

Step-by-step explanation:

I say so

What is the range & domain of the set
R: {(-6, 14), (10,19), (4, -9), (3, 2), (6, -13)}

Answers

Answer:

Domain { -6,3,4,6,10}

The range is the output values, listed from smallest to largest with no repeats

Range { -13,-9,2,14,19}

Step-by-step explanation:

The domain is the input values, listed from smallest to largest with no repeats

Domain { -6,3,4,6,10}

The range is the output values, listed from smallest to largest with no repeats

Range { -13,-9,2,14,19}

Answer:

Range: 14, 19, -9, 2, -13

Domain: -6, 10, 4, 3, 6

Step-by-step explanation:

I don't know but this is it I think .

There are 6 people named A,B,C,D,E,F. The people named A,B, and C are all over the age of 40. The people named D,E,F are all under the age of 40. How many different orders are there for the people to sit on a bench, if both ends of the bench must be occupied by someone over the age of 40?

Answers

Which of the following statements is generally true about change in the workplace ? a ) Most people accept change easily . b) Smart companies can avoid change altogether. c) Change in the workplace fairly infrequently d) Individuals can learn to manage the change in their lives.

A tank is filled at a constant rate. 10 minutes after filling is started, the tank contains 4.8L of water. After 35 minutes the tank contains 7.3L of water.

a. Find the rate at which the tank is being filled?

b. Find the initial volume of fluid in the tank and express it as a function in terms of V and t.

c. Find how long it takes to filled, if the tank has a maximum capacity of 60L?

Answers

Answer:

Part A)

0.1 liters per minute.

Part B)

There was initially 3.8 liters of water.

[tex]\displaystyle V(t) = 0.1(t - 10) + 4.8[/tex]

Part C)

562 minutes.

Step-by-step explanation:

A tank is filled at a constant rate. After 10 minutes, the tank contains 4.8 L of water and after 35 minutes, the tank contains 7.3 L of water.

Part A)

We can represent the current data with two points: (10, 4.8) and (35, 7.3). The x-coordinate is measured in minutes since the tank began to be filled and the y-coordinate is measured in how full the tank is in liters.

To find the rate at which the tank is being filled, find the slope between the two points:

[tex]\displaystyle m = \frac{\Delta y}{\Delta x} = \frac{(7.3)-(4.8)}{(35)-(10)} = \frac{2.5}{25} = 0.1[/tex]

In other words, the rate at which the tank is being filled is 0.1 liters per minute.

Part B)

To find the function of the volume of the tank, we can use the point-slope form to first find its equation:

[tex]\displaystyle y - y_1 = m( x - x_1)[/tex]

Where m is the slope/rate of change and (x₁, y₁) is a point.

We will substitute 0.1 for m and let (10, 4.8) be the point. Hence:

[tex]\displaystyle y - (4.8) = 0.1(x - 10)[/tex]

Simplify:

[tex]\displaystyle y = 0.1(x-10) + 4.8[/tex]

Since y represent how full the tank is and x represent the time in minutes since the tank began to be filled, we can substitute y for V(t) and x for t. Thus, our function is:

[tex]\displaystyle V(t) = 0.1(t - 10) + 4.8[/tex]

The initial volume is when t = 0. Evaluate:

[tex]\displaystyle V(0) = 0.1 ((0) - 10) + 4.8 = 3.8[/tex]

There was initially 3.8 liters of water.

Part C)

To find how long it will take for the tank to be completely filled given its maximum capacity of 60 liters, we can let V(t) = 60 and solve for t. Hence:

[tex]60 = 0.1(t - 10) + 4.8[/tex]

Subtract:

[tex]55.2 = 0.1(t - 10)[/tex]

Divide:

[tex]552 = t - 10[/tex]

Add. Therefore:

[tex]t = 562\text{ minutes}[/tex]

It will take 562 minutes for the tank to be completely filled.

PLEASE HELP!!!! It’s urgent

Answers

Answer:

(-1,4)

Step-by-step explanation:

The interval in which the function is decreasing is (-1, 4)

Answer:

The domain of a function is the set of all possible inputs for the function.

Using the table provided, the set of all possible inputs is the interval [-6 ; 4].

The range of a function is the complete set of all possible resulting values of the dependent variable (y, usually), after we have substituted the domain. In plain English, the definition means: The range is the resulting y-values we get after substituting all the possible x-values.

Using the table provided, the range is estimated on the interval [-10;20]

The y-intercept is the value on the Y-axis where the function crosses the Y-axis.

Using the table provided, the function crosses the Y-axis for f(0)=18 so for the value y = 18 in the table.

The x-intercept is the value on the X-axis where the function crosses the X-axis.

It happens twice, for f(-6)=0 and f(3)=0.

We estimate the Maximum to be 20, and the Minimum -10.

The function is positive over the interval [-6, 3], and negative over (3;4]

The function is decreasing approximately at f(-1)=20 so at the estimated interval (-1;4]

8 A test rocket is fired and follows a path described by y = 0.1x(200 – x). The height is y metres above
ground and x is the horizontal distance in metres.
How far does the rocket travel horizontally?
b How high does the rocket reach mid-flight?

Answers

Answer:

a) The rocket travels 200 meters horizontally.

b) The height of the rocket mid-flight is of 1000 meters.

Step-by-step explanation:

Height of the rocket:

The height of the rocket, in meters, after an horizontal distance of x, is given by:

[tex]y = 0.1x(200 - x)[/tex]

a) How far does the rocket travel horizontally?

This is x when [tex]y = 0[/tex]. So

[tex]0.1x(200 - x) = 0[/tex]

Then

[tex]0.1x = 0[/tex]

[tex]x = 0[/tex]

And

[tex]200 - x = 0[/tex]

[tex]x = 200[/tex]

So

The rocket travels 200 meters horizontally.

b How high does the rocket reach mid-flight?

This it the height y when x = 0, so:

[tex]y = 20*100 - 0.1*100^2 = 1000[/tex]

The height of the rocket mid-flight is of 1000 meters.

If sam runs 63,756 feet in 70 min what's his miles per hour

Answers

Answer:

10.35 miles per hr

Step-by-step explanation:

first convert ft into mi

then calculate the distance traveled in one min

multiply the answer by 60

then you get the answer

You decide to go on a 4 day backpacking trip. The first day you walk 8 miles at northeast, on the second day, you walk 4 miles at eastsouth, and on the third day you walk 3 miles at southwest. On the fourth day you need to head straight back to your car. How far do you have to walk, and in what direction

Answers

Answer:5

Step-by-step explanation:

Where the above parameters are given,  you need to walk a distance of approximately √41 miles back to your car.

How to compute the above

To calculate the total distance you need to walk, you can use the Pythagorean theorem since you have a right triangle formed by the north and east displacements.

Distance = √((Distance north)² + (Distance east)²)

= √((5 miles)² + (4 miles)²)

= √(25 miles + 16 miles)

= √41 miles

Hence, you need to walk a distance of  approximately √41 miles back to your car.

As for the direction, based on the net displacements, you are 5 miles north and 4 miles east of your car, so the direction would be a combination of north and east, often referred to as northeast.

Learn more about distance  at:

https://brainly.com/question/26046491

#SPJ2

Please help! The question is in the image

Answers

Answer:

I am pretty sure that your answer would be 3.

Step-by-step explanation:

The reason why is because if B if half of line segment AD and AD is equal to 12, then B must be equal to 6 since half of 12 is 6. Next, since C is the mid-point for line segment BD then C must be 3 since half of 6 is 3. And finally, that means line segment BC is three since it is 1/2 of BD.

Hope this helps! :)

Answer:

BC = 3

Step-by-step explanation:

If B is the midpoint of AD, that means AB = BD

AD = 12 so BD = 1/2 of AD  and BD = 6

If C is the midpoint of BD, that means BC = CD

BD = 6 so BC = 1/2 of BD  and BC = 3

Other Questions
Throughout Act 2 it is implied that Eliza values herself and has self-respect. All of the following provides evidence for that conclusion except...she is suspicious of Higgins offer and his wager with Pickeringshe has put ostrich feathers in her cap and tried to clean her coatshe claims that despite not having a mom she is a good girlshe stays when Higgins offers her chocolate and taxi rides HELP ILL HIVE BRAINLIEST A(n) ________ is a cone of debris that forms where an ephemeral stream emerges from the confines of the canyon. Its runoff spreads over the gentler slopes at the base of the mountains and quickly loses velocity, dumping most of its sediment load within a short distance. indicate the world that does not belong to the sound group bean,pick,bead,peak 9.2% written as a decimal is if you meet any of the famous activist who works for child rights, what question would you ask? prepare a list of questionaries Which statement best describes the limitations of Progressivism?Progressives expressed little interest in trustbusting.Progressive reforms failed to address civil rights for all citizens.Women were prevented from participating in the Progressive movement. A tank is filled at a constant rate. 10 minutes after filling is started, the tank contains 4.8L of water. After 35 minutes the tank contains 7.3L of water. a. Find the rate at which the tank is being filled?b. Find the initial volume of fluid in the tank and express it as a function in terms of V and t.c. Find how long it takes to filled, if the tank has a maximum capacity of 60L? 20. simplify each of the following: see the above pictureand get 40 points II. Choose the word whose stressed pattern is pronounced differently from the others.1. A. improve B. support C. educate D. invent2. A. patient B. become C. quality D. science3. A. unable B. impolite C. unhealthy D. imbalance4. A. enormous B. discover C. benefit D. inventor make a list of principle of lever. During the data transmission there are chances that the data bits in the frame might get corrupted. This will require the sender to re-transmit the frame and hence it will increase the re-transmission overhead. By considering the scenarios given below, you have to choose whether the packets should be encapsulated in a single frame or multiple frames in order to minimize the re-transmission overhead.Justify your answer with one valid reason for both the scenarios given below.Scenario A: Suppose you are using a network which is very prone to errors.Scenario B: Suppose you are using a network with high reliability and accuracy. 13. Is the word LAWYER in the following sentence a predicate nominative or apredicate adjective?My sister became a lawyer after years of study. Please help A. SASB. AASC. HAD. LLE. ASAF. HL hi! would it be correct if i said"the components of our solar system consists of our star, the Sun, and everything bound to it by gravity" I'm not looking for punctuation corrections but just if it sounds right or something like that :> A rectangle has a length of 8ft and a width of 9ft. What is the area of the rectangle, in ft^2? What features are typically associated with postmodern fiction? a colleague needs to move a 500 pound object that's on a lever. the fulcrum is 6 inches from the object. If your colleague weighs 180 pounds how far will your colleague need to be from the fulcrum to move the object? Use the formula B = (P x A) W, and round your answer to two decimal places. If two angles are complementary, find the measure of each of angle. Greg has a list of payments for goods brought every month for the years 2009,2010,2011, and 2012. He needs information from only the last three years. He can filter this by clicking on